9. What is the value of x?

9. What Is The Value Of X?

Answers

Answer 1

Answer:

A, 61.

Step-by-step explanation:

(2x+24). If we sub in 61 for x, we get (2(61)+24). Follow order of operations.

2 times 61= 122. Add 24 and you get 146. We can know this is correct because we are given the other angle and it's degrees, whch is also 146 degrees. Therefore, the answer is A, 61.


Related Questions

Given g(x)=−23(x−4)2+3, g ( x ) = − 2 3 ( x − 4 ) 2 + 3 , what is the value of g(–5)? *Type in your answer. g(–5) =

Answers

The output value of g( -5 ) in the function g( x ) = −23( x - 4 )2 + 3 is 417.

What is the output value of g(-5) in the given function?

A function is simply a relationship that maps one input to one output, that is, each x-value can only have one y-value.

Given the data in the question;

g( x ) = −23( x - 4 )2 + 3g( -5 ) = ?

To find the g( - 5 ), replace all the occurrence of x with -5 in the function and simplify.

g( x ) = −23( x - 4 )2 + 3

g( -5 ) = −23( -5 - 4 )2 + 3

g( -5 ) = −23( -9 )2 + 3

g( -5 ) = −23( -9 × 2) + 3

g( -5 ) = −23( -18) + 3

g( -5 ) = ( -23 × -18) + 3

g( -5 ) = ( 414 ) + 3

g( -5 ) = 414 + 3

g( -5 ) = 417

Therefore, the output value of g( -5 ) is 417.

Learn more about functions here: brainly.com/question/2541698

#SPJ1

write an equation of the line below.​

Answers

Answer:

y = 2x + 3

Step-by-step explanation:

The y-intercept is where x = 0. Therefore, the y-intercept would be 3. To find the slope, you do Δy/Δx (Δ = Change In). To find that, you go up 4 times, then move to the right 2 times to reach the second dot. Now, you do 4 / 2, which equals 2. Your final equation will be y = 2x + 3.

PLEASE ANSWER QUICK IT IS THE LAST PROBLEM ON HERE AND IT IS DUE TODAY!!!

Answer should be inequality!

Ray had scores of 75, 82, 94, and 77 on his first four algebra tests. What must he score on the next test to have an average of at least 85?

Please show work as well if possible.

Answers

Answer:

x ≥ 97

Step-by-step explanation:


Average = Sum of values divided by the amount of numbers in a data set


Data Set: {75, 82, 94, 77, ?}

75 + 82 + 94 + 77 + ? = ? ÷ 5 = 85

85 x 5 = 425

75 + 82 + 94 + 77 + ? = 425 ÷ 5 = 85

75 + 82 + 94 + 77 = 328


425 - 328 = 97

Ray must score greater than or equal to 97, or x ≥ 97 in order to have an average of at least 85.

find the volume of the rectangular prism. need an integer or decimal.

Answers

We will have the following:

[tex]V=(\frac{9ft\ast9ft}{2})(12ft)\Rightarrow V=486ft^3[/tex]

So, the volume is 486 ft^3.

Hi I just need Part A. I’m in high school calculus 1 and this is a homework. thank you !

Answers

SOLUTION

Given the question in the image, the following are the solution steps to answer the question.

STEP 1: Define what happens when a function is shrunk vertically

When the factor is greater than 1, the condition that satisfies the function is to multiply by 1.4

When the graph is verticcally shrunk, this means that the function is multiplied by 1.4

Hence, the function becomes:

[tex]1.4\times f(x)=1.4\cdot f(x)[/tex]

STEP 2: Define what happens when the graph is shifted left by 3 units.

If the function is shifted left by 3 units, this implies that we add 3 units to the x of the function.

This gives:

[tex]1.4.f(x+3)[/tex]

Please helppp as fast as possible 20 points ***
FIND THE DOMAIN & RANGE OF THE FUNCTION
g(x)=|x + 4|

Please helppp as fast as possible

Answers

Answer:

Domain: (−∞,∞),{x | x ∈ R}

Range: [0,∞),{y|y≥0}

Step-by-step explanation:

Solve using substitution.
X = 7
-2x 9y = 13
(__ , __)?
Submit

Answers

Answer:

(7,-3)

Step-by-step explanation:

x = 7

-2x - 9y = 13

* Substituition is the easieat way to solve such a function.

- Firstly, put "7" instead of "x" in the equation.

- multiply "7" by "-2"

- Solve for "y"

* -2x-9y = 13

* -2×7-9y = 13

* -14-9y = 13

* -9y = 27

* -9y/-9 = 27/-9

* y = -3

# Since x = 7, the output/"y" will be -3

(7,-3)

The larger of two numbers is five times the difference of the small number and one. If sum of the two numbers is 50 what are the two numbers?

Answers

Answer:

Step-by-step explanation:

40.833 recurring and 9.166 recurring

or 40 5/6 and 9 1/6

Call the small number 's'

Call the big number 'b'

The word problem says:

B+S=50 and also that 5(S-1)=B which multiplied out is 5S-5=B

So, 5S-5=B=50-S

Therefore 6S=55 and the rest history!

Answer:

Step-by-step explanation:

this is a little hard for me help me out please

Answers

The simplified form of the expression as a fraction is -43/390.

Solving expressions and simplifying fractions

Fractions are expression written as a ratio of two integers. For instance, a/b are known as fractions.

Given the expression below:

[2-|-2/3-2(-1/5)|]÷(-13)

In order to solve the expression, we will use the PEMDAS rule to have:

[2-|-2/3-2(-1/5)|]÷(-13)

[2-|-2/3+1/10)|]÷(-13)

Find the LCM

[2-|-20+3/30|]÷(-13)

[2-|-17/30|]÷(-13)

[2-(17/30)]÷(-13)

[60-17/30]÷(-13)

43/30÷(-13)

Change the division sign to multiplication

43/30÷(-13)

43/30 * -1/13

-43/390

This gives the simplified form of the expression.

Learn more on simplifying expressions here: https://brainly.com/question/28036586

#SPJ1

What is the image point of (-7,3) after the transformation rx-axis o R180º?

Answers

[tex]r_{x-axis}\circ R_{180}[/tex]

In a composite transformation as given you make first the transformation in the right and then the transformation in the left.

For the given point: (-7, 3)

1. Rotation 180°

[tex]\begin{gathered} P(x,y)\rightarrow P^{\prime}(-x,-y) \\ \\ P(-7,3)\rightarrow P^{\prime}(7,-3) \end{gathered}[/tex]

2. Reflection over x-axis:

[tex]\begin{gathered} P^{\prime}(x,y)\rightarrow P^{\prime}^{\prime}(x,-y) \\ \\ P^{\prime}(7,-3)\rightarrow P^{\doubleprime}(7,3) \end{gathered}[/tex]Then, the image of given point after the composite transformation is (7,3)

vAt her Fourth of July party, Sally serves her famous grilled chicken. Her secret is to use 6 fluid ounces of lemon-garlic marinade for every pound of chicken. Sally plans to grill 4 pounds of chicken. How many cups of marinade should she use

Answers

In linear equation, 3 number of cups will be needed to marinade 4 pounds of chicken for grill.

What are a definition and an example of a linear equation?

An equation with only one variable is referred to as a linear equation in one variable. It has the mathematical formula Ax + B = 0, where A and B can be any two real numbers, and x is an unknowable variable with just one possible value. A linear equation in one variable would be 9x + 78 = 18, for instance.

Here we have the data from the question:-

Marcy will use 6 fluid ounces of lemon-garlic marinade for every pound of chicken.

Marcy plans to grill 4 pounds of chicken.

So total marinade required will be:-

Total marinade  =6x4=24 ounces

Now we know that the conversion:-

1 fluid Ounce = 0.125 Cup

24 fluid ounce =0.125 Cup

Hence 3 number of cups will be needed to marinade 4 pounds of chicken for grill.

Learn more about linear equation

brainly.com/question/11897796

#SPJ10

The roots of the quadratic function are -2 and -6.Which of the following are the two factors of the quadratic expression?X + 2X-2X - 6X + 6X + 8X - 8

Answers

roots = {-2, -6}

Factors: (x + 6) and (x + 2)

The equation for line s can be written as x - y = -2. Line t is perpendicular to lines and passes through (-4, 1). What is the equation of line t?​

Answers

Answer:  x + y = -3

=========================================================

Explanation:

Anything perpendicular to Ax+By = C is of the form Bx-Ay = D

The given equation is x-y = -2 showing that A = 1, B = -1 and C = -2.

So Bx-Ay = D will update to -x-y = D

Plug in the coordinates of (-4,1) to find D.

-x-y = D

D = -x-y

D = -(-4)-1

D = 4-1

D = 3

We go from -x-y = D to -x - y = 3

Then multiply both sides by -1 to end up with x + y = -3

Find the area of 2x-9 + x+5

Answers

=3x - 4
Hope this helps :)

Answer: 3x-4

Step-by-step explanation: First Simplify, add the numbers 2x-9+x+5=2x-4+x Then combine like terms, 2x-4+x= 3x-4

can you explain each step please for i can write it on a paper along with you

Answers

The two groups are given one is those who are studied and other is those who are not studied.

Studied - 88,100,94,79,92,100,95,83,89,99,100,91,89,95,100,93,96,84.

Arrange in order.

79, 83, 84, 88, 89, 89, 91, 92, 93, 94, 95, 95, 96, 99, 100, 100, 100, 100

The mean of this group is

[tex]\frac{88+100+94+79+92+100+95+83+89+99+100+91+89+95+100+93+96+84}{18}[/tex][tex]\frac{1667}{18}=92.6[/tex]

The mean is 92.6.

The median of this group is

Use the median formula for even

[tex]m=\frac{(\frac{n}{2})^{th}+(\frac{n}{2}+1)^{th}^{}}{2}[/tex]

Substitute the value of n =18

[tex]m=\frac{(\frac{18}{2})^{th}+(\frac{18}{2}+1)^{th}^{}}{2}=\frac{9^{th}+10^{th}^{}}{2}[/tex][tex]m=\frac{93+94}{2}=93.5[/tex]

The median is 93.5.

The mode of this group is 100 as it is appears 3 times .

The another group is not studied group.

Not studied - 82,72,45,91,58,83,65,87,90,77,73,89.

Arrange in order-

45, 58, 65, 72, 73, 77, 82, 83, 87, 89, 90, 91

The mean is determined as

[tex]\frac{82+72+45+91+58+83+65+87+90+77+73+89}{12}=\frac{912}{12}=76[/tex]

The median is determined as

[tex]m=\frac{(\frac{n}{2})^{th}+(\frac{n}{2}+1)^{th}}{2}[/tex]

Substitute n=12.

[tex]m=\frac{(\frac{12}{2})^{th}+(\frac{12}{2}+1)^{th}^{}}{2}=\frac{6^{th}+7^{th}}{2}=\frac{77+82}{2}=79.5[/tex]

There is no mode.

So, from the all the data given.

The mean of the group that studied is over 15 percent points higher than the mean of the group that did not studied.

[tex]92.61-76=16.61[/tex]

The median of the group that did not studied is less than 80.

In general , those students that studied scored much higher than those students that did not study.

The correct options are a , c and e.

HELP ME PLEOPLE THIS IS SO CONFUSING HELP ME I WILL GET SUSPENDED IF I DON"T ANSWER THIS HELP ME PLEASE HELP I WILL GET A 0 AND SUSPENDED HELP

Answers

D and B ! Your welcome!

Please assist Math with 50 points!

Answers

Answer:

A. (1/2) bucket

Step-by-step explanation:

    6           11

4 ------ - 3 ------- = ?

    14          12

14 × 4 = 56

56 + 6 = 62

12 × -3 = -36

-36 - 11 = -47

 62       47

------- - -------

  14        12

 62(12)        47(14)

-------    -     -------

  14(12)         12(14)

744     658       86

------- - ------- = --------

 168     168       168

 86 ÷ 2

-------

168 ÷ 2

43

----- = 0.511

84

0.511 ≈ (1/2)

I hope this helps!

Answer:

1/2 bucket

Step-by-step explanation:

Solve the inequality and graph the solution on the line provided.
3x+17_< 41

Answers

So if your problem is: 3x + 17 (less than or equal to sign) 41

Your answer should be: x _< 8

(The “<“ is on top of the underline btw)

The sign between x & 8, should be: “less than or equal to” sign.

And your point on the graph should go form right to left. Starting on right side “8”, then proceeding to the negative side

Check the graph below that I provided


How do we calculate slope when given two points on a coordinate plane?

Answers

Step-by-step explanation:

Given the coordinates of two points on a line, use the slope formula to find the slope of the line. The slope formula is m=(y2-y1)/(x2-x1) or the change in y value versus the change in x value. The coordinates of the first point represent x1 and y1. The coordinates of the second point are x2, y2.

Answer:

To find the slope, use the formula Rise over Run.

1.) Insert the Y coordinates into their correct space.

2.) Plug in the X coordinates.

3.) Subtract your Y coordinates.

4.) Subtract your X coordinates.

5.) Reduce if needed.

6.) You're done!

What is the value of logx?y3lodwhen given the following:Zlog(x) = 3log(y)=2log(z)= -1

Answers

Given the question

[tex]\log (\frac{x^2y^3}{z})[/tex]

To resolve this, we can follow the steps below

Step1: Apply the logarithm rule

[tex]\log (\frac{x^2y^3}{z})=\log x^2+\log y^3-\log z[/tex]

This will give=>

[tex]\log x^2+\log y^3-\log z=2\log x+3\log y-\log z[/tex]

Since we have been given that

[tex]\begin{gathered} \log x=3 \\ \log y=2 \\ \log z=-1 \end{gathered}[/tex]

Step2: Substitute the given values into the equation

[tex]2\log x+3\log y-\log z=2(3)+3(2)-(-1)[/tex]

=>

[tex]6+6+1=13[/tex]

Answer = 13

Chad doesnt have any money, but he owes his sister $15, owes his brother $9 and owes his friend $24. What integer could be used to represent the amount of money chad has?

Answers

Answer:

- 48.00

Step-by-step explanation:

Add all the debt up.

Answer: -48

Step-by-Step Explanation:

Money that Chad owes his Sister = $15

Money that Chad owes his Brother = $9

Money that Chad owes his Friend = $24

Total Money he owes = 15 + 9 + 24 = $48

Hence, Total Money he has = -48 (since he doesn't have any money)

a potter forms a piece of clay into a right circular cylinder. as she rolls it, the height of the cylinder increases and the radius decreases. assume that no clay is lost in the process. suppose the height of the cylinder is increasing by centimeters per second. what is the rate at which the radius is changing when the radius is centimeters and the height is centimeters?

Answers

The radius of the cylinder is decreasing at the rate of 0.22 cm per second.

One of the most important and fundamental curvilinear of a geometric shapes, a right circular cylinder has historically been a three-dimensional solid.

It is regarded as a prism with a circle as its base in basic geometry. In several contemporary fields of geometry and topology, a cylinder can alternatively be characterized as an infinitely curved surface.The radius of the cylinder is the length of the line joining the circumference and the center of the circle.The derivative of the logarithm,  dx /dt, is also the growth rate. The fractional change, must be used to represent a minor change in the logarithm. if the variable grows at the constant fixed rate g.

Volume of a right circular cylinder = π r² h

Now we differentiate w.r.t to get :

[tex]\frac{dV}{dT} =2 \pi r h\frac{dr}{dt} + \pi r^2\frac{dh}{dt}[/tex]

Taking the values we get :

[tex]-34.3 = 154 \frac{dr}{dt}[/tex]

[tex]\frac{dr}{dt} = -0.22[/tex]

Hence the radius is decreasing at the rate of -0.22 cm per second.

To learn more about cylinder visit:

https://brainly.com/question/16134180

#SPJ4

si cada 8.33 minutos me dan 10 dolares cuantos dolares tendria en 6 horas ademas cuantos dolares ganaria cada 30 minutos

Answers

The payment for each 6 h is equal to $ 432.2 while the payment for each 30 min would be  $ 36.01.

Rule of Three

It's a math tool for solving problems about proportion. You can relate more than 2 variables and from the proportion, it's possible to find an unknown number.

Conversion Unit Time

In the International System Units (SI), the standard unit for time is the second (s).  For solving any exercise, you need to know the relation between the units of the time. See below:

1h=60 min = 3600s

1 min = 60 s

Now you can solve the question from the rule of three for finding the payment in dollars.

The question gives: an each 8.33 min, the payment is 10 dollars. Thus, from this relation you can write:

For 6 h

First, you should convert 6h to minutes, then,

1 h ----- 60 min

6h ----- x

x=60*6=360 min

After that, you should apply the rule of three for finding the payment in dollars.

   8.33 min     -----  10 dollars.

   360 min    -------  x

8.33x=360*10

8.33x=3600

x= $ 432.2

For 30min

You should apply the rule of three for finding the payment in dollars.

   8.33 min     -----  10 dollars.

   30 min    -------  x

8.33x=30*10

8.33x=300

x= $ 36.01

Read more about the rule of three here:

brainly.com/question/24514897

#SPJ1

Sandra purchased $1,200 worth of electronic equipment on her new credit card, with an annual interest rate of 14.99%. If she has no other charges on the card and does not pay off the balance at the end of the month, how much money will she owe the credit card company after 1 month? Estimate the interest using the monthly periodic rate.

Answers

Given data:

The given amount of equipment purchased by the Sandra is A=$1,200.

Choose a relationship model that will reach $0 in the same number of days as this scenario: mika has $12 and spends $2 each day. y = â€""2x â€"" 12 y = â€""3x 18 y = â€""4x 12 y = â€""12x 2

Answers

The equation that represent the relationship model that will reach $0 in the same number of days as the scenario gave is: y = -2x + 12

To solve this problem, we have to state the equation using the information of the problem.

Information about the problem:

Mika has = $12Mika spend = $2 per dayDay = xy = $0

Writing the equation according to the information gave, we have:

y = -2x + 12

Where:

y = 0$x = days

So that (y) equals zero, the number of days (x) should be = 6

y = -2x + 12

0 = -2*6 + 12

0 = -12 + 12

0 = 0

The "y" variable is the dependent variable, because it will change according to the number of days passed (x).

The "x" variable is the independent variable, because it doesn't depend on any other value, just the number of days passed.

What is an equation?

An equation is the equality between two algebraic expressions, which have at least one unknown or variable.

Learn more about equation at: brainly.com/question/2972832 and brainly.com/question/27815607

#SPJ4

I have a graph with the question. If the parent function is y= 2 to the power of x plus 2, which is the function of the graph

Answers

The parent function of the graph is

[tex]y=2^x[/tex]

The function of the grapg will be

[tex]y=2^{x\text{ + a}}\text{ + b}[/tex]

at x equals infinity y equals 1

Therefore

[tex]\begin{gathered} \text{x =}\infty\text{ then y = 1} \\ \text{this implies} \\ b\text{ = 1} \end{gathered}[/tex]

Therefore the function will now be

[tex]y=2^{x\text{ + a}}\text{ + 1}[/tex]

from the graph

x = 2 when y = 2

Therefore,

[tex]\begin{gathered} 2=2^{2\text{ + a}}\text{ + 1} \\ 2-1=2^{2\text{ + a}} \\ 1=2^{2\text{ + a}} \\ \text{recall 1 = 2}^0 \\ 2^0=2^{2\text{ + a}} \\ 2\text{ + a = 0} \\ a\text{ = - 2} \end{gathered}[/tex]

Therefore, the function is

[tex]y=2^{x\text{ - 2}}\text{ + 1}[/tex]

48/8 6/w

1=?



Help me please helpp

Answers

Answer:

w = 1

Step-by-step explanation:

You have to divide 48 by 8 to get 6, so you have to divide 8 by 8 to find w.

8/8 = 1

someone include steps if you can

Answers

Using the concept of the equation, it can be concluded that Levi answered the first question wrong and the last two correctly. Thus,

No

Yes

Yes

How to check whether the value of x satisfies the equation?

To check the value of x satisfies the equation, put the value of x and if it matches the resultant, it is correct.

Now solving the first part, x/4=20;x=5

So, putting x=5 in the equation 5/4=1.25 is not matching with the resultant ie. 20.

Therefore, this is the incorrect value of x.

Now solving the second part, 1.5x-8=7;x=10

So, putting x=10 in equation 1.5*10-8= 15-8=7

It matches with the resultant.

Therefore, this is the correct value of x.

Now solving the third part, 4x+6=58; x=13

So, putting x=13 in equation 4(13)+6= 52+6=58

It matches with the resultant.

Therefore, this is the correct value of x.

To know more about equations, visit:

https://brainly.com/question/10413253

#SPJ13

If ax-3=bx+5 then the solution for x in terms of a and b is

Answers

In this equation the solution for x in terms of a and b is 8.

What is equation?

There are many different ways to define an equation. The definition of an equation in algebra is a mathematical statement that demonstrates the equality of two mathematical expressions. For instance, the equation 3x + 5 = 14 consists of the two expressions 3x + 5 and 14, which are separated by the 'equal' sign. Mathematical algebraic equations typically have one or more variables.

Sol- As per the question given that- ax-3= bx+5------(equation 1)

Let ax=3-----(equation 2)

And bx =5------(equation 3)

On adding equation(1)and (2) we get

(ax)+(bx)= 3+5

(ax)+(bx)=8

x(a+b)+y(a+b)=8

(x+y)+(a+b)=8

1×(a+b)=8

Thus

a+b=8

To know more about equation click -

https://brainly.com/question/2972832

#SPJ13

PLEASE HELP!!!!!!!!! It’s algebra

Answers

Answer:

See below

Step-by-step explanation:

From 5 to 10 mugs is 5 mugs  

   These cost  110 - 67.50 = 42.50

        So each mug costs 42.50 / 5 = 8.50 each  

              With a 'base cost' of 25 dollars

   (Base cost might be artwork, design, order processing, shipping or whatever.....but this amount is added to each order)

(As a check    20 mugs would be   25 + 20 (8.50) = 195   <====yep

150 mugs will then be 25   +  150 ( 8.50) = 1300 dollars

Other Questions
Aside from cell membrane, where else can you find a bilayer of lipid? Water distribution causes many issues in africa. What is the majority of fresh water in africa used for?. 2(x + 7) = 6x + 9 - 4x need help assap look at file attached What value is most important to your family? How do you display it in your everyday life?What is your greatest talent? How did you discover it?Do you think you could go for one month without using the Internet? Why or why not? What would be the hardest part?In your opinion, is it important for people to study history? Why or why not?Write a letter to your future self.use one pargraph for each question The graph of f(x) is shown in black.Write an equation in terms of f(x) to match the redgraph.For example, try something like this:f(x)+3, f (x - 2), or 4f(x). Which feature in microsoft word gives you the option to increase or decrease the indentation of items in a bulleted list or numbered list?. A chemical company mixes pure water with their premium antifreeze solution to create an inexpensive antifreeze mixture. The premium antifreeze solution contains 65%pure antifreeze. The company wants to obtain 260 gallons of a mixture that contains 45% pure antifreeze. How many gallons of water and how many gallons of the premium antifreeze solution must be What is the average rate of change from f(-1) to f(1)?Type the numerical value for your answer as a whole number, decimal or fractionMake sure answers are completely simplified question 18:Evaluate: summation from n equals 2 to 8 of 12 times 4 tenths to the n plus 1 power period Round to the nearest hundredth. (1 point) the marginal propensity to consume is: group of answer choices total consumption in a given period divided by total disposable income the fraction of each additional dollar of disposable income spent on consumption the percentage of total disposable income spent on consumption that part of the consumer dollar that goes to the purchase of investment goods I need help part two and three of this question:A line passes through the following points: (6,3) and (2,9)1. Write the equation of the lineWhich I got y=-3/2 x+122. Write an equation of a line that is perpendicular to the original form. 3. Write the equation of a line that is parallel to the original form. ABCD is a rectangle. Find the length of AC and the measures of a and f. which of the following is true of the landrum-griffin act? a. it requires unions to establish bylaws and make financial reports. b. it prohibits employees from engaging in protected concerted activities. c. it prohibits employers from asking employees to sign yellow dog contracts. hello! i need help on this question and the (select) questions have the options of 1997 to 2006 Solve for w. 3w + 2w - 3w = 8 if an unforeseeable event occurs which makes it unreasonably difficult or expensive for one party to perform the contract, the contract might be discharged on the grounds of: fitness is a measure of reproductive success. if an individual with genotype (bb) produces 6 offspring, and a heterozygous individual (bb) produces 10 offspring, and a third individual with genotype (bb) produces 5 offspring, what is the fitness value for the individual with (bb) genotype? Which logical argument could you use to prove that triangle ABC is congruent to triangle DEF?A. SSS PostulateB. HL TheoremC. SAS PostulateD. ASA Postulate Match each equation to one of the lines5x + 3y = 20